Középiskolai Matematikai és Fizikai Lapok
Informatika rovattal
Kiadja a MATFUND Alapítvány
Már regisztráltál?
Új vendég vagy?

Fórum: GEOMETRIA

  [1]    [2]    [3]    [4]    [5]    [6]    [7]    [8]    [9]    [10]    [11]    [12]    [13]    [14]    [15]    [16]    [17]    [18]    [19]    [20]    [21]    [22]    [23]    [24]    [25]    [26]    [27]    [28]    [29]    [30]    [31]    [32]    [33]    [34]    [35]    [36]    [37]    [38]    [39]    [40]    [41]    [42]    [43]    [44]    [45]    [46]    [47]    [48]    [49]    [50]    [51]    [52]    [53]    [54]    [55]    [56]    [57]    [58]    [59]    [60]    [61]    [62]    [63]    [64]    [65]    [66]    [67]    [68]    [69]    [70]    [71]    [72]    [73]    [74]    [75]    [76]    [77]    [78]  

Szeretnél hozzászólni? Jelentkezz be.
[1818] Fálesz Mihály2014-01-31 14:31:09

A Bolyai-féle szögharmadolás:

Előzmény: [1817] jonas, 2014-01-31 10:28:25
[1817] jonas2014-01-31 10:28:25

Ez egy érdekes kérdés, tetszik. Valamiért azt gondoltam elsőre, hogy mindig meg lehet szerkeszteni az összes metszéspontot, de most már nem vagyok biztos benne.

Előzmény: [1816] Sinobi, 2014-01-30 09:54:23
[1816] Sinobi2014-01-30 09:54:23

Ha adott egy kupszelet es egy kor es 0/1/2/3 metszespontjuk, akkor mely esetekben szerkesztheto meg a tobbi metszespont, es mely esetekben nem?

Előzmény: [1786] HoA, 2014-01-05 18:54:23
[1815] Loiscenter2014-01-24 20:25:57

köszi - tanulmányozom a magyarazatot!!!

Előzmény: [1814] w, 2014-01-24 13:42:00
[1814] w2014-01-24 13:42:00

Igen. Még arra sincs szükség, hogy A és B azonos sebességgel mozogjon.

Elég azokat az időpontokat vizsgálni, melyekben az A golyó az eredeti A0 pontban van. Tegyük fel, hogy ameddig A elmegy B0-ba és vissza A0-ba, annyi idő alatt B s.I utat tesz meg, ahol I a félkörív hossza. Célunk az, hogy belássuk, hogy tetszőleges a>0-hoz van olyan n pozitív egész, melyre ns kisebb, mint a távolságra van egy egész számtól.

Ha s racionális szám, akkor nincs mit bizonyítani, létezik olyan n>0 egész, melyre ns egész szám lesz.

Ha s irracionális, akkor pedig tekintsünk egy hatalmas nagy N-et, és az \left[\frac{i}{N};\frac{i+1}N\right) intervallumokat i=0,1,...,N-1-re. Mivel s irracionális, ezért {ns} (ns törtrésze) minden n>0-ra más és más értéket vesz fel. Ezért az {ns} számokat minden n=1,2,...,N+1-re véve, ez az N+1 szám benne van előbbi intervallumok egyikében. Skatulya-elv szerint így lesz két n, n1 és n2, melyre {n1s} és {n2s} ugyanabban az intervallumban van, eltérésük <N-1. Másfelől mivel triviálisan \left\{\{n_1s\}-\{n_2s\}\right\}=\{(n_1-n_2)s\}, ezért kaptunk egy olyan n=n1-n2 számot, melyre {ns}<N-1, vagyis |n|s egy egész számtól kisebb, mint N-1 távolságra van. Márpedig N-1 tetszőlegesen kicsi lehet. (A Kronecker-tételre is hivatkozhattunk volna.)

Összességében tetszőlegesen kis ívhossz lehet A és B távolsága, amiből következik az állítás.

Előzmény: [1813] Loiscenter, 2014-01-23 23:25:25
[1813] Loiscenter2014-01-23 23:25:25
[1812] w2014-01-16 13:06:53

Szép és elemi kérdés, a bizonyítás is nagyon szép, a vélemény szerintem helytálló. A probléma egyébként az IMO 1991/5. feladat.

Persze lehetne vitatkozni, bőven vannak még geometriai szépségek, például ez egészen szép és egyszerű, ám ránézésre egyáltalán nem triviális tétel.

Előzmény: [1811] Loiscenter, 2014-01-16 11:13:32
[1811] Loiscenter2014-01-16 11:13:32

Vietnami matematikusok önkéntes versenyérol( legszebb és legelemibb bizonyitás) idézem egy feladatot :

Az ABC háromszög belsejében van egy P pont. bizonyitando hogy a PAB , PBC, PCA szögek között van 30° -nál nem nagyobb!

kivancsi vagyok a véleményetekre!

[1810] w2014-01-12 23:35:37

Igazad van, nem voltam teljesen alapos, de nem nehéz kipontosítani (látószögek...).

Előzmény: [1809] Loiscenter, 2014-01-12 23:17:35
[1809] Loiscenter2014-01-12 23:17:35

Az állitás hogy ABX vagy ABY közé irható kör sugara nagyobb mint ABC -énél - ezt nem látom!!!

Előzmény: [1808] w, 2014-01-12 21:41:09
[1808] w2014-01-12 21:41:09

Tekintsük a legnagyobb olyan kört, amely a sokszög valamely három csúcsa köré írható, a három csúcs legyen A,B,C. Belátjuk, hogy ez tartalmazza az egész sokszöget, és három szomszédos csúcson halad át.

Ha ez a kör minden csúcsot tartalmaz, akkor egyben tartalmazza az egész sokszöget. Ezért ha nem tartalmazza az egész sokszöget, akkor van olyan X csúcs, amelyet a kör nem tartalmaz. De ekkor ABX kör nagyobb sugarú lesz az ABC körnél, ami ellentmond választásunknak.

Tegyük fel, hogy nincs olyan három szomszédos csúcs, amelyre a köréjük írt kör sugara maximális. Ekkor az előbb igazoltak szerint van, mondjuk körüljárás szerint A és B között, egy olyan Y csúcs, amely a legnagyobb körön belül található. De ekkor az ABY kör sugara ismét nagyobb lesz, mint ABC sugara, ez ellentmondás.

Előzmény: [1807] Loiscenter, 2014-01-12 18:55:23
[1807] Loiscenter2014-01-12 18:55:23

Egy konvex sokszögnél mindig megtalalható 3 egymast követö csucsot, melyekre a köré irható kör tartalmazza az egész sokszöget?( elemi egyszerü bizonyitas jó lenne)

[1806] epsilon2014-01-12 16:29:01

Köszi a linket Kemény Legény! Igen, az összegnél elírtam, kösz a javítást, de utólag láttam, a sejtésem úgy sem talál Üdv: epsilon

Előzmény: [1804] Kemény Legény, 2014-01-12 11:04:39
[1805] fityfiritty2014-01-12 13:04:51

És egy apróság: \frac{n (n + 1)}2 \ne \frac{(n - 2)(n - 1)}2 .

Előzmény: [1803] epsilon, 2014-01-11 16:13:13
[1804] Kemény Legény2014-01-12 11:04:39

Az alábbi linken van pár bizonyítás a háromszöges esetre: itt.

Előzmény: [1803] epsilon, 2014-01-11 16:13:13
[1803] epsilon2014-01-11 16:13:13

Ha nem tévedek, a kérdésre a válasz az alábbi lenne, de nem tudom bizonyítani, sem a tetraéderre átültetni :-(

Előzmény: [1802] epsilon, 2014-01-11 15:17:36
[1802] epsilon2014-01-11 15:17:36

Üdv mindenkinek! A felvetett hasáb számlálási feladat kapcsán előkerült egy másik feladat is: Egy szabályos háromszög minden oldalát osszuk fel n egyenlő részre, majd ezeken át húzzunk az oldalakkal párhuzamos egyeneseket. Így a háromszöget n×n kis kongruens háromszögre osztottuk. Összesen hány háromszög látható az ábrán? Sajnos az eredményre zárt alakban nincs tippem. Olyan megoldás érdekelne, amelyik kiterjeszthető a térbe is, ha a szabályos háromszög helyett szabályos tetraédert tekintünk, és azt bontjuk kis tetraéderekre. Tudna-e valaki segíteni? Előre is köszönöm, üdv: epsilon

Előzmény: [1796] Kemény Legény, 2014-01-08 10:23:09
[1801] HoA2014-01-09 12:38:28

Rajzoljunk a két szögszár teljes egyenesére egy-egy „centiméter” skálát. Azokat a pontpárokat összekötve, melyek távolságának összege a metszésponttól egy adott ( előjeles ) érték, Steiner II. tételének duálisa szerint szintén egy másodrendű görbe érintőit kapjuk, melyek között a végtelen távoli egyenes (mint a megfelelő pontokat összekötő egyenes) is ott van. Ekkor a burkolt görbe parabola.

A feladat feltételeinek megfelelő, a beírt kör O középpontján áthaladó, a szögfelezőre szimmetrikus egyenespárt tehát akár [1799] szerint O-ból húzott hiperbola érintőként, akár az itteni parabola érintőiként megrajzolhatjuk. Az [1800] –ban felvetett egzisztencia kérdés persze vizsgálandó: Kívül esik-e O az érintendő görbéken?

Előzmény: [1800] marcius8, 2014-01-09 12:30:07
[1800] marcius82014-01-09 12:30:07

Csak egy felvetés:

1. Van-e olyan egyenes, amely átmegy a háromszög beírt körének a középpontján és felezi a háromszög területét? Ekkor ez az egyenes nyilván felezi a háromszög kerületét is.

2. Van-e olyan egyenes, amely átmegy a háromszög beírt körének a középpontján és felezi a háromszög kerületét? Ekkor ez az egyenes nyilván felezi a háromszög területét is.

A probléma általánosítása: Adott síkbeli zárt görbéhez (sokszög) keressünk olyan egyenest, amely felezi az adott zárt görbe által meghatározott síkidom területét és kerületét

Előzmény: [1791] w, 2014-01-07 20:59:13
[1799] Sinobi2014-01-08 21:01:11

nem mondtam, hogy elemi. Lenyegeben ugyanaz, mint marcius8-e,

terulet felezes tetszoleges pontbol:

Be lehet latni, hogy ha adott egy szogszar, akkor azok az egyenesek, amelyek egy rogzitett T teruletnyi darabot vagnak le belole, egy hiperbolat fognak erinteni. (abbol, hogy egy hiperbolanak minden pontja az aszimptotatol valo tavolsagok szorzata allando, kovetkezik, hogy a levagott haromszog terulete konstans?) tehat a feladat nem mas, mint hogy adott egy hiperbola (aszimptotakkal, es 3 darab/tetszoleges szamu erintojevel, szerkesszunk hozza egy ponton at erintot. Ezt meg meg lehet csinalni.

Előzmény: [1797] Loiscenter, 2014-01-08 11:49:24
[1798] epsilon2014-01-08 15:46:43

Köszi Kemény Legény! Ilyen elgondolás alapján írtam azt a képletet, csak nem voltam biztos abban, hogy választások során biztos-e, hogy mind hasáb lesz. Még egyszer köszi, üdv: epsilon

Előzmény: [1796] Kemény Legény, 2014-01-08 10:23:09
[1797] Loiscenter2014-01-08 11:49:24

Melyikre tudod elemi uton megoldást?

Előzmény: [1794] Sinobi, 2014-01-08 03:06:00
[1796] Kemény Legény2014-01-08 10:23:09

A válasz helyesnek tűnik. Számoljuk meg a hasábokat a testátlóik alapján! Válasszunk ki egy tetszőleges csúcsot (ezt (n+1)3-féleképpen lehet), majd az átellenes csúcsot is (ezt már csak n3-féleképpen lehet, mert nem lehet egy síkban/oszlopban az előző ponttal). Ezzel minden hasáb minden testátlóját (kétszer is) megszámoltuk, minden hasábot 8-szor találtunk meg (4 testátlója van egy hasábnak + az átellenes csúcsokat fordítva is figyelembe vettük). Azaz a végeredmény \frac{(n+1)^3\cdot n^3}{8}, ami épp a képen látható kifejezéssel egyenlő.

Előzmény: [1795] epsilon, 2014-01-08 10:00:31
[1795] epsilon2014-01-08 10:00:31

Boldog Új Évet Kívánok Mindenkinek! Az új esztendőben előkerült egy régi feladatom, amire csak sejtem az eredményt, a bizonyítást nem tudom. Beírom ide, mert biztos vagyok, hogy valaki tud segíteni. A feladat: Egy kockát az oldalakkal párhuzamos síkokkal felosztunk n×n×n egyforma kiskockára. Hány hasáb látható az így keletkezett felosztáson? Sejtésem szerint annyi amennyi a beszúrt képen látható. Tudna valaki segíteni a bizonyításban? Előre is köszönöm, Üdv: epsilon

[1794] Sinobi2014-01-08 03:06:00

ki van zarva, hogy ugyanolyan nehezek legyenek, mert az egyikre tudom a megoldast, a masikra nem :)

Előzmény: [1793] Loiscenter, 2014-01-07 23:31:04

  [1]    [2]    [3]    [4]    [5]    [6]    [7]    [8]    [9]    [10]    [11]    [12]    [13]    [14]    [15]    [16]    [17]    [18]    [19]    [20]    [21]    [22]    [23]    [24]    [25]    [26]    [27]    [28]    [29]    [30]    [31]    [32]    [33]    [34]    [35]    [36]    [37]    [38]    [39]    [40]    [41]    [42]    [43]    [44]    [45]    [46]    [47]    [48]    [49]    [50]    [51]    [52]    [53]    [54]    [55]    [56]    [57]    [58]    [59]    [60]    [61]    [62]    [63]    [64]    [65]    [66]    [67]    [68]    [69]    [70]    [71]    [72]    [73]    [74]    [75]    [76]    [77]    [78]